Διαιρετότητα με το 1989

Συντονιστές: cretanman, silouan, rek2

Άβαταρ μέλους
matha
Γενικός Συντονιστής
Δημοσιεύσεις: 6423
Εγγραφή: Παρ Μάιος 21, 2010 7:40 pm
Τοποθεσία: Θεσσαλονίκη

Διαιρετότητα με το 1989

#1

Μη αναγνωσμένη δημοσίευση από matha » Τετ Νοέμ 13, 2019 6:18 pm

Αν \displaystyle{n\geq 3} είναι ακέραιος, να αποδείξετε ότι

\displaystyle{1989|\Bigleft (n^{n^{n^n}}-n^{n^{n}}\Bigright)}


Μάγκος Θάνος

Λέξεις Κλειδιά:
Άβαταρ μέλους
ΦΩΤΙΑΔΗΣ ΠΡΟΔΡΟΜΟΣ
Δημοσιεύσεις: 921
Εγγραφή: Πέμ Νοέμ 22, 2018 9:43 pm

Re: Διαιρετότητα με το 1989

#2

Μη αναγνωσμένη δημοσίευση από ΦΩΤΙΑΔΗΣ ΠΡΟΔΡΟΜΟΣ » Τετ Νοέμ 13, 2019 11:55 pm

Καλησπέρα!

Δεν είμαι σίγουρος για την ορθότητα της λύσης:

Γράφουμε k=n^{n^{n^{n}}}-n^{n^{n}}=n^{n^{n}}\left ( n^{\left (n^{n^n}-n^n \right )} -1\right )
Είναι 1989=3^2\cdot 13\cdot 17,αρκεί 9,13,17 \mid k
Δουλεύουμε για κάθε ένα ξεχωριστά:
Αν  n \not\equiv 0 (\mod 13) αρκεί n^{\left (n^{n^n}-n^n \right )} -1\equiv 0(\mod13)\Leftrightarrow n^{n^n}-n^n\equiv 0(\mod(ord_{13}n))
Όμοια πρέπει n^{n^n}-n^n\equiv 0(\mod(ord_{17}n)),n^{n^n}-n^n\equiv 0(\mod(ord_{9}n)).
Όμως για :
 n( \mod 13 )\in \left [ 1,16 \right ] είναι ord_{13}n=\left ( 1,2,3,4,6,12 \right )

n (\mod 17)\in \left [ 1,16 \right ] είναι ord_{17}n=\left ( 4,8,16 \right )
n (\mod 9)\in \left [ 1,8 \right ] είναι ord_{9}n=(2,3,6)
Επομένως αρκεί n^{n^n}-n^n\equiv 0 (\mod 16),n^{n^n}-n^n\equiv 0(\mod3)
Το δεύτερο δείχνεται εύκολα,για το πρώτο πήρα όλες τις περιπτώσεις και δουλεύει(ίσως το αναλύσω αύριο αυτό).


ΠΑΠΑΔΟΠΟΥΛΟΣ ΣΤΑΥΡΟΣ
Δημοσιεύσεις: 3600
Εγγραφή: Πέμ Φεβ 27, 2014 9:05 am
Τοποθεσία: ΧΑΛΚΙΔΑ- ΑΘΗΝΑ-ΚΡΗΤΗ

Re: Διαιρετότητα με το 1989

#3

Μη αναγνωσμένη δημοσίευση από ΠΑΠΑΔΟΠΟΥΛΟΣ ΣΤΑΥΡΟΣ » Σάβ Νοέμ 16, 2019 1:05 pm

Για να μην γράφω θα χρησιμοποιήσω αυτούσια κομμάτια της απόδειξης του Πρόδρομου .
Επίσης θα θεωρήσω αυτονόητα τα προφανή.
Π.χ αν n=0mod 13
τότε ο αριθμός διαιρείται με 13

Γράφουμε k=n^{n^{n^{n}}}-n^{n^{n}}=n^{n^{n}}\left ( n^{\left (n^{n^n}-n^n \right )} -1\right )

Είναι 1989=3^2\cdot 13\cdot 17,αρκεί 9,13,17 \mid k

Αρκεί να δείξουμε ότι ο αριθμός

{\left (n^{n^n}-n^n \right )}=n^n(n^{n^{n}-n}-1) είναι πολλαπλάσιο του 3 και 16

(Euler \varphi (17)=16,\varphi (13)=12,\varphi (9)=6)

Γιαυτό αρκεί ο n^{n^{n}-n}-1

να είναι πολλαπλάσιο του 3 και του 8
(Euler \varphi (16)=8)

Για το 3 είναι προφανές.

Για το 16 μπορούμε να υποθέσουμε ότι ο n είναι περιττός .

Αν n=2k+1 τότε

(1+2k)^{1+2k}-(2k+1)=1+(2k+1)2k+\frac{1}{2}(2k+1)2k(2k)^{2}+8r-(2k+1)=
8r+4k^{2}(1+k(2k+1))=8m
και η απόδειξη ολοκληρώθηκε, παραλείποντας βέβαια κάποια σχεδόν ''προφανή'' κομμάτια.


Απάντηση

Επιστροφή σε “Θεωρία Αριθμών - Επίπεδο Αρχιμήδη (Seniors)”

Μέλη σε σύνδεση

Μέλη σε αυτήν τη Δ. Συζήτηση: Δεν υπάρχουν εγγεγραμμένα μέλη και 2 επισκέπτες